Xem bài viết đơn
Old 11-07-2012, 01:36 AM   #2
Harry Potter
+Thành Viên+
 
Harry Potter's Avatar
 
Tham gia ngày: Sep 2008
Bài gởi: 32
Thanks: 24
Thanked 26 Times in 6 Posts
Dùng cái này là xong mà $(a_k+1)=\left(a_k+\frac 1{k-1}+\cdots+\frac 1{k-1}\right)\geq k\sqrt[k]{\frac{a_k}{(k-1)^{k-1}}}$
[RIGHT][I][B]Nguồn: MathScope.ORG[/B][/I][/RIGHT]
 
Harry Potter is offline   Trả Lời Với Trích Dẫn
The Following 5 Users Say Thank You to Harry Potter For This Useful Post:
baotram (11-07-2012), hoang_kkk (13-07-2012), kien10a1 (11-07-2012), nghiepdu-socap (11-07-2012), ngocson_dhsp (11-07-2012)
 
[page compression: 8.19 k/9.23 k (11.28%)]